« first day (2549 days earlier)      last day (2469 days later) » 

12:04 AM
@EricSilva I think I'm going again for dynamics on Friday
It starts off by talking on Riemannian manifolds so everyone's just like yo
Wat?
 
12:20 AM
What is the topic
 
Hyperbolic sets and $\epsilon$-orbits
 
@AkivaWeinberger that was already hashed out and discussed way more than needed.
@kthonenice have you proved that the equation has only one solution?
you have yet to prove that the elementary matrices are invertible
yet you use it without justification
 
@Typhon he probably proved it way before he asked this question
Or at least we can safely assume that, it's easy anyway, just undo the process
 
is $\emptyset\cup\{a\} = \{a,\{\emptyset\}\}$?
 
it is never safe to assume anything
 
12:35 AM
Well then you'll have a bit of a problem
 
he needs to prove that there is a unique solution and that the elementary matrices are invertible
 
Because whenever you answer a stack question at all, you have to make sure that this person has proven the requisite linear algebra results and whatnot
 
@Daminark you can only assume what is given to you and what is previously proven.
he hasn't stated a proof that the equation has a unique solution
my point is that there might be more than one A
 
My point is that he's probably already done that proof in class. Many classes do ask when they do a computation to manually verify on a particular example some previously proven general theorem
At least that's not a point worth fixating on
For now
 
Linear Algebra courses don't ask people to prove stuff like that...
In fact, now I wonder if matrix equations can have multiple solutions.
 
12:39 AM
I mean, they might not ask you to prove it on a homework assignment necessarily, but they either did the proof in class, have it in the book and are expected to read it, or the class just takes the statement as granted. In any of those three cases, let it be
It depends on the rank of the matrix
 
@Daminark No, you don't get it. The class I was in literally never mentioned the issue of the uniqueness of A. I'm cranking my gears thinking back and AFAIK it was never once brought up (vectors were, but not matrices). So now I'm curious if I can even construct an equation with infinite A.
it might be that I'm asking him to prove something as trivial as 4k= 8 having a unique k. Obviously it does. It's just... assumed.
XD
now I feel like an idiot.
 
I mean, take an elementary row operation, you can undo it. So once you show that an elementary operation has a matrix representation (do it to the identity), you know that is invertible
 
Use the fact that the empty set is, by definition, the set with no members @heather
 
So then given a matrix $A$ and a sequence of operations that lead them to $B$, you'll recover $A$ uniquely by undoing
 
@skullpatrol, right...nvm, that was a dumb question.
 
12:42 AM
what I'm thinking though is that for non-invertible E's A is still unique.
or unique in the weird case that A simply doesn't exist.
 
Anyway, I've got a lecture to prep so I should tab out now. Though last thing, what new row operations would you be defining then? And I guess, what would having those operations get you?
With that, peace out!
 
Cya
 
@Daminark I wasn't saying that they would be row operations. I was just thinking a random arbitrary matrix equation EA = I.
where E is not invertible
im claiming that A is either unique or non-existent.
wait I am so stupid
that's the same issue with units in number theory
if A exists than there exists another element E such that A * E = I
Therefore, E must be the inverse of A.
Therefore E is invertible.
Therefore, if E is not invertible than A is not invertible and the equation is contradicted.
I'm literally trying to claim an absurd case that is proved false by what I am given. XD
 
Right, Demonark. All elementary matrices are invertible. But often we even multiply them by non-square matrices!
 
$\sum_{i=1}^{10}x_{i}\equiv 0 (mod11)$
if I had that congruence, how would I estimate how many how many numbers satisfy it?
 
12:59 AM
Hi prof @TedShifrin
 
 
2 hours later…
2:41 AM
Is it possible for two circles to meet a a single point?
 
@user400188 it'd be a figure 8
 
one usually says that the two circles are tangent in that case.
and on that note, this is a fun picture: en.wikipedia.org/wiki/Descartes%27_theorem#/media/…
 
Interesting. I assume that when they meet the tangent at that point is the same for both?
 
Well, yes.
They wouldn't be meeting if it weren't the same point for both :)
 
I meant same tangent for both.
 
2:55 AM
ah. in any case, yes.
 
I wonder if its possible to make two curves meet at one point (but not cross) and have different tangents at that point. My guess is its not possible (at least if the curve is smooth and differentiable).
 
yeah, definitely not.
 
3:23 AM
hi chat
 
3:34 AM
what is the current greatest upper bound on the smallest number not know to be prime or composite?
 
Hey @skullpatrol
 
@AkivaWeinberger wait... I only need the statement: "If a + bx > c + dx, then a > c or b > d". I am pretty sure that is all I need. Can it be proven in general? If you can prove it in general then by all means do so!
on another note, I think I can prove it when the "b" in x^2 + ax + b = 0 is negative. However, that is just a thought.
 
assume c>a and d>a
 
The only way one line can lie above another for all $x$ is if they're parallel.
 
@shaihorowitz huh?
 
3:41 AM
So that requires $b=d$. Once you've got that, having $a>c$ will enforce the equality for all $x$.
 
...
@Semiclassical wat
 
@Daminark Your Very Secret Number Theory Study Group is slowly getting to 14 days of inactivity (last post here 11d ago) - at 14 days it might be frozen.
 
@Typhon what's the domain that $x$ is in? If it should work for $x=0$, that forces $a>c$ immediately
 
if x then y, if not y then not x. not y = c>a and d>a show it implies not x
 
...
im well aware how to prove such a statement. The issue is that I'm fairly certain it is impossible for negative x.
also, I was replying to a comment from earlier
 
3:44 AM
ah much more useful then huh
 
@shaihorowitz why are you trying to prove it?
 
ah, you're not interested in all $x$?
 
@Martin thank you for letting me know!
 
I'm not interested in anything. Akiva is.
 
why is chat suddenly so exclusive?
 
3:45 AM
I've already been proving this stuff. I've been challenging Akiva.
XD
@shaihorowitz It's not. It's just i wasn't asking for help. Nor did i need it. I was sharing a proposition I had already proved to see if akiva could do it. I didn't know you were trying to help me.
 
i get it now were all clear
perhaps you could help me any idea on the smallest number not known to be prime or composite?
 
1 is not prime or composite, and every other number is prime or composite :P
 
@shaihorowitz that's really just a computation issue. Brute force is the easiest method there. By definition something is either prime, composite, or a unit. There's only two integer units. So... I'd use google to find the largest unchecked number.
 
@Daminark Is it ok to joint that room even if I do not plan to actively read the book, only occasionally look at how the reading is going, perhaps with posting a comment here and there?
 
Wouldn't that be the largest prime number known plus 1.
 
3:48 AM
@TheRaidersofLasVegas no
 
@Martin absolutely!
 
thats what ive been doing, its just so scattered,
 
@TheRaiders that'd be even
 
he says not known to be composite or prime
 
1 is a cheeky answer
 
3:49 AM
depends
did you mean the largest number to be a unit or the largest number to be categorized into the set prime or composite?
as in... is 34345345455556359 prime or composite?
 
I mean I was doing it because "prime or composite" is, if you're being a jerk in interpretation, means one, or the other, or both
 
its not what i'm looking for in any case though. and it depends on the defenition of known the fact that it can work is why i label it as cheeky
 
@shaihorowitz ah I see. Well... just use google. It's really just a computation check.
 
The only excluded numbers are units, aka $1$ and $-1$
 
^^
 
3:51 AM
But yeah I mean, the largest known prime is like, $2^{\text{Holy crap}} - 1$
 
@shaihorowitz btw, the negation of a > c or b > d is a <= c and b <= d
 
Where "Holy crap" is prime @Daminark
 
if we assume the integers are positive, then the issue is the sign of x.
if x is negative....
PANIC
 
and interestlngly we dont know if theres a prime of the form 2^holy crap -1 less then 2^Holy crap -1,
 
3:53 AM
Yeah. And it's like fucked as well, 70 million+
 
@shaihorowitz uummm.... 2^n - 1 form primes are all over the place aren't they?
127 is a prime, right?
....right?
 
theres only 43 or so known
 
oh
 
conjectured an infinite amount
 
I'm thinking of x^2 + 1 primes. XD
 
3:54 AM
Yeah 127 is prime
 
128 = 2^8
 
but we dont know if there are any between the 40th know and 43rd known
 
Such as... the 41st or 42nd?
 
well we dont know the number of primes of 2^n-1 between the 43rd known and 40th known.
 
Ah
That makes more sense lmao
 
3:57 AM
yeah gotta get my grammar right
 
True dat
 
user84215
Because I think it is not good that only a room owner can create an event for the room, I have created the room, Discussing Specific Topics . In this room, people can discuss the topics they have specified before. Each topic lasts in this room for at most one day; It depends on its popularity among others. Its duration also can be specified before beginning.
 
now primes of the form $2^(2^(n)+1$ is conjectured to be a finite set
the first few are prime
 
4:12 AM
yeah, Fermat primes
No one has a proof that there aren't any more past the fifth, but we're up to 292 composite Fermat numbers.
 
I never liked the misnomer Fermat's last theorem.
 
I mean now it is a theorem
 
user84215
He claimed that he had proved it
 
^^^^^^^^^
precisely
it had been proven by fermat
the proof was lost
 
And who knows, maybe the reason it couldn't fit was that he had to build up all of algebraic geometry to do so and just didn't have space in the margin!
@Typhon well, he allegedly had it, though the proof that people think he had in mind is false
 
user84215
4:20 AM
But there were not enough space to write it in his notebook.
 
I read somewhere once that he had a proof and showed it to collegues and that people had confirmed it was true
the proof was lost
 
That sounds dubious.
 
shift-6
 
Even if that story were true, all it would indicate is that he had a proof that convinced his colleagues.
It doesn't follow that that it's an argument that would hold up to modern inspection.
 
user84215
4:23 AM
Some great mathematician like Lagrange also claimed that they had proved Euclid's parallel postulate.
 
Gauss said that about a lot of his "theorems."
 
@aminliverpool except their peers laughed it off.
@Semiclassical of course. But Fermat's Last Theorem was true, was it not?
 
Sure. That's also entirely irrelevant.
 
is it?
the point is that we don't know what he tried.
 
If you take two people who have convincing arguments that fail to be proofs for both a statement and its negation
 
4:24 AM
so he could've been correct.
@Daminark the difference though is that modern inspection shows who was right
 
@Typhon I think the point was that he only "could" have been correct, he wasn't necessarily correct
 
user84215
I mean great mathematicians also can make mistakes in their works.
 
I'd also point out that the modern proof is waaaaaaay beyond what Fermat would have known.
 
So we don't know if Fermat had a legit proof or not on basis of the stuff. There's a suspect which is convincing but false, and unless he's devilishly clever to either come up with an elementary argument or develop the theory of modular elliptic curves single-handedly (only the former would've been remotely possible :P), there's not yet good reason to believe his proof worked
 
there's a hell of a lot of daylight between "Fermat's last theorem is provably true" and "Fermat could have proven it using the mathematics of his era"
 
4:27 AM
true
but my point is that naming it theorem was reasonable
because people in the time period believed it
 
Ehhhh
 
People of which time period?
 
the time period that it was written in, obviously
 
user84215
Perhaps he found an interesting elementary proof, as he said, that has never come into other mathematicians' minds.
 
4:28 AM
^^^
 
Perhaps. But I wouldn't bet money on it.
 
me neither
but does it matter?
 
If anything, I'd wager a pretty fair bet in the other direction.
 
Define: proof
 
@TheRaiders any system of assertations with a QED or a box at the end
 
4:29 AM
Also, though, this line of reasoning is under the assumption that he had an argument which could convince others. Is there any historical evidence of such?
 
if a man convinces 100000 mathematicians that they've proven the Riemann Hypothesis and nobody can debate it, is there any reason to not rename it the Riemann Theorem?
 
:-D @Daminark
 
That's the thing. You probably wouldn't call Riemann's theorem.
What would probably happen is that it'd be given the name of whoever gave the actual proof.
 
@Semiclassical I know I read it somewhere reputable once. Don't recall where. I've always taken it as a "Library of Alexandria" type situation. The proof was lost.
 
(...in principle, anyways. in practice, Arnold's principle applies)
 
4:31 AM
and of course, nobody could replicate it
perhaps fermat only had a sketch of a proof?
either way
he somehow convinced people it was a theorem and not a conjecture
 
Some historical discussion here: mathoverflow.net/questions/33265/…
 
Unrelated to the issue I was giving akiva earlier, here is another proof you guys can help me with. Suppose x is a negative irrational number. Prove that for integers a, b, c, and d that if a + bx > c + dx, then a > c or b > d.
The issue here is that I've tried proof by contradiction and gotten to where I have a <= c and b <= d. The issue is that multiplying by the latter gives bx <= dx.... and I don't think that can be used to form a contradiction.
 
Thanks for the link @Semiclassical
 
The stated inequality is equivalent to $(b-d)x>c-a$. If $b>d$ then this means that $x>\frac{c-a}{b-d}$ which is inconsistent with $x$ being negative.
 
ooooooh of course!
genius
a > c or b < d
 
4:35 AM
So what you really want to show is that $a+bx>c+dx$ for any negative irrational $x$ implies $a>c$.
 
i think that is too strong
if it were true, then I could use that to simplify a lot of my proofs
like.... eliminate a lot of needless writing
 
Well. There are three cases of relevance. Either $b>d,b=d,$ or $b<d$.
Oh, derp. I made an error.
 
what?
 
Rest in gg no re
 
If $b>d$ but $c-a<0$, then $\frac{c-a}{b-d}<0$.
In which case that inequality only shows that $x$ can't be too negative.
 
4:38 AM
'too' negative?
oh i see
 
Example: If $(a,b,c,d)=(4,3,2,1)$ then that becomes $x>\frac{2-4}{3-a}=-1$
 
ah
 
Which forbids $x=-\sqrt{2}$ but not $x=-1/\sqrt{2}$
 
of course
but wait
that forbids the case of a > c and b > d, does it not?
 
That it does.
 
4:42 AM
but the case a > c and b < d is always true
 
Hm. If "a>c and b>d" is false, but "a>c and b<d" is true
 
a > c or b < d appears trivially true
 
Something seems fishy.
 
?
 
Not sure what.
I kinda want to think about this geometrically.
i.e. what pairs of lines are consistent with "x is some negative irrational number"
 
4:47 AM
a + bx > c + dx; Proof that a > c or b < d

Suppose not. Then, a <= c and b >= d. Multiplying the latter equation by x we get bx <= dx. Then, adding the equations together we get a + bx <= c + dx. This is a contradiction. Therefore the statement is true. []
 
Hmm. You're using the fact that x<0 to argue that b>=d implies bx <= dx. That seems sound.
Note that you didn't need the fact that x is irrational in there, though that's not terribly shocking.
 
@Semiclassical basically I proved that if for Z[x] where x is a positive real irrational and there exists a unit y that is neither 1 nor -1, then there exists a unit w such that all units are powers of w or the negatives of powers of w.
the thing I'm doing now is to build up for the case that x is negative.
 
Right.
 
complex will come later...
like.... way later
 
lol
Hmm. My attempt at a direct proof.
 
4:51 AM
for the statement I proved about inequality?
 
Right.
 
oh ok
fair enough
 
If b<d, then the conclusion is true. If b=d, then a>c follows immediately. If b>d, then the inequality becomes x>(c-a)/(b-d). But x<0, so this is only true when a>c.
 
really the linchpin for my proof is to show that if a + bx > c + dx (and they are units), then a > c
or show that b < d
either way, that proof forces them to become a discrete set
 
Right.
 
4:54 AM
I can essentially show there is an element succeeding 1
 
I think the proof by contradiction is slick enough.
 
and then from there I can use contradiction to show there isn't one between 1 and that unit y
as since y > 1 it's powers diverge to infinity
therefore all other potential units will be within two powers of y
divide by the lesser power and you get that there is some other unit between 1 and y
contradiction
[]
 
Neat.
 
it works for any real ones
technically I handwaved away the proof that there exists a y^n and y^n+1 such that the potential unit is between them
but.... the professor I had the original course said "it was quite clever" and "needed real analysis to patch an error". I didn't understand the real analysis so I just take that as granted for the sake of sanity.
i figure anyone can trivially patch that
tbf, I don't claim uniqueness
just that they exist
I mean heck....
 
I'm not in a position to judge either way.
 
4:58 AM
I say that every quadratic has two solutions if they are irrational
yet I've never proved it nor intend to anytime soon
 
About the only mathematical 'discovery' I can ever claim to have made in number theory is a rule to find remainders mod 7, and that's a silly little thing.
 
@Semiclassical It was an introductory abstract proofing course that happened to be number theory themed in one of the homeworks for an advanced extra project for special credit. So when the professor said, "you need real analysis" he was saying it more in the sense of: "how in the world did you think of this?"
 
lol, nice.
 
and since I didn't understand his fix...
I don't need to use it. It's true so I can just appeal to previous knowlegde.
probably a property of discrete sets anyways
:p
anyways, now I'm proving that if the norms are equal then the integer components must be unequal in the way I specified
that essentially complete the proof barring minor corrective steps in making my other proofs more generic
i kept saying "let x be positive"
but in some cases that is no longer needed
 
Nice.
 
5:03 AM
of course then comes the issue of when is x positive and negative
but....
that's arguable a different issue
and of course there's also the issue of whether nontrivial units even exist in certain sets
this merely goes under the impression of existence
 
5:24 AM
x^2 + ax + b = 0 for integers a and b. Both values of x are negative.

Definition of norm/given info:

N(c + dx) = c^2 - cda + bd^2 = 1
N(e + fx) = 1
c + dx > e + fx
c, d, e, and f are positive integers.

Prove:

c > e and d < f

Proof:

Suppose c <= e. Then d < f. Multiplying together and squaring we get that c^2 <= e^2 and cd < ef. Since both values of x are negative, we know that a is a positive number and b is negative number as if x and x' are the two solutions of the polynomial we can say that a = -x - x' and xx' = b. Therefore, cda < efa and c^2 <= e^2 and bd^2 < bf^2.
@Semiclassical
im stuck
d < f i think produces the same issue
the issue is that both solutions are negative
hrmm
you know what?
who cares
i'll work on these cases later on
for now I have a case when x^2 + b = 0
and im happy with it
 
6:01 AM
hi. can sb help me w this?
there are 4 people we want to group them into groups of 4, we do this twice, means each is assigned to a group twice. so how many ways can we assign these people?
actually lets say there are 4 teachers to be assigned to teach in 4 schools and each school needs 2 teachers. now that makes sense !
I'd say we have $ 4 ! $ conditions for the first time, and then once again we do it and there is another $ 4! $ so as the answer I'd get $ 4! ^2 $
is that right?
 
6:21 AM
that would be correct if you allow situations where you have the same assignment twice
 
@s.harp what do you mean?
 
lets say you want to assign a,b,c,d to 1,2,3,4 and to 1,2,3,4
 
ok
 
if you allow (a-> 1, a->1) (b->2, b->2) (c->3, c->3) (d->4, d->4) you are correct
of course in general the two target sets are not the same, and the statement that aou have the same assignment twice does not make sense
 
0
Q: Suppose that x^2 = b is an irrational positive real number. Show that there exists a nontrivial unit in Z[x].

TyphonSuppose that $Z[x]$ is composed of all numbers of the form $c + dx$ where $c$ and d are integers and $x$ is an irrational positive solution to the polynomial $x^2 + b = 0$ where $b$ is an integer. I seek to show that there exist any elements that are units aside from $1$ and $-1$. I know that...

im not sticking around but I know this has to be true
i see no reason why it isn't
I'm looking for a somewhat simple proof though
nothing too complicated
 
6:26 AM
@parvin What he means is that in this case you'd be assigning a teacher, say Alice to both faculty positions in the same school.
 
@s.harp @TimTheEnchanter you mean what I'm doing is ONE condition among n conditions it could be? like, alice twice to 2nd sch and petre to 2nd sch, but no other choices is considered..? is that so ?
edit :
alice to 2nd school and petre to 2nd school or alice twice to 2nd school (only one of the conditions)
 
@parvin I'm saying that here you are counting the aforementioned scenario, which is not part of the configurations to be counted
 
is not or is?
I guess I'm considering only one of the conditions this way, isn't it?
I know i'm wrong i just wanna know why
 
@parvin You have counted all of the solutions, the problem is you've counted extra invalid ones as well.
Unless it is possible for one teacher to have both jobs in the same school, in which case It is right.
 
what I was trying to do was this:
S1 -> {T,T,T,T}
S2 -> {T,T,T}
S3 -> {T,T}
S4 -> {T}

S1_T {T,T,T,T}
S2_T {T,T,T}
S3_T {T,T}
S4_T{T}

so I thought I'd right 4! * 4!
it seams a little weird ... i'm confused
 
6:41 AM
@parvin I don't understand what that represents
 
I mean for the first time each of the schools have accepted a teacher and for 2nd time each are accepting another
 
@parvin So the same teacher can fill both faculty positions in a single school?
 
yes yes
 
@parvin Then it's correct, that's what we have been saying all this time.
 
7:11 AM
ok ! :) @TimTheEnchanter
 
@parvin Have a nice day :)
 
7:40 AM
Some time ago I think at least I read that the geometric series converges everywhere on it's radius of convergence except for "+1". Is there a question about it? Could find anything...
 
@Felix.C Hmm, I recall it being fairly easy to show
 
@TobiasKildetoft Actually what I said was wrong, obviously "-1" does not converge either...but I don0t how to proceed with complex numbers...
 
Right, I just realized that
In fact, it does not converge on any root of unity
 
yes..
 
Does it in fact converge anywhere on the radius?
Hmm, might help to write it in terms of sin and cos
 
7:54 AM
I'll try...thanks
 
 
1 hour later…
9:01 AM
hi guys, let $\omega_j$ a sequence such that $$
\sum_{j=0}^{+\infty} \omega_j \le \infty
$$ and for all $k$ we have $$
\omega_k \leq \sum_{j=k+1}^{+\infty} \omega_j
$$
and assume (but you can prove it is true) that for al $x$ in a certain interval $[-A,A]$ we have
$$
x = \sum_{j=0}^{+\infty} d_j \omega_j \;\;, d_j \in \left\{-1,1\right\}
$$
my question is it true that if I pick $x,y$ in such interval with $x < y$ then there's a $j_0$ such that $d_{x,j} = d_{y,j}$ for $j < j_0$ and $d_{x,j} < d_{y,j}$ for $j = j_0$
namely
if given $x$ and is expansion
and I get the sequence $d_j$
is the order preserved?
my guess would be yes, because the binary expansion is a specific case of this one
 
user84215
9:25 AM
Why don't you suggest your ideas for starting a new event in Discussing Specific Topics in order to discuss a specific math subject?
 
10:48 AM
Hi, $$\text{Calculate with approximation of }10^{-3} \text{, } \{(2+\sqrt 5)^{2^{2017}}\} \text{ with } \{1.53\}=0.53$$
 
Oh God
I'm guessing it'll end up being zero
Actually, revised guess: It'll end up being one
(That is, >0.999)
 
I'm betting on $\pi$ because the solution of a problem is always one of $0,1,\pi$ and Akiva already took the first two
 
Yes but Why @AkivaWeinberger
 
:P
@Dattier Because a similar thing happens with the golden ratio
and also I tested out the first few examples on my calculator and it seems to work
 
And why
There are an explination
 
10:56 AM
Hm, so it's a root of $x^2-4x-1$
This makes me want to think about the integer Fibonacci-like sequence $G_{n+2}-4G_{n+1}-G_n=0$
or $G_n=4G_{n-1}+G_{n-2}$
Clearly $G_n:=(2+\sqrt5)^n+(2-\sqrt5)^n$ satisfies this recurrence.
 
For indice you can say that, for $$\{(3+\sqrt 5)^{2^{2017}}\}$$
 

« first day (2549 days earlier)      last day (2469 days later) »